1
answer
0
watching
64
views
10 Jun 2018

Give a function Y = AK^0.4 L^0.6

A. if K increase by 10% what is the percentage change in Y, MPK, and MPL?

B. if L increase by 10% what is the percentage change in Y, MPK, and MPL?

c. Is there evidence of diminishing marginal productivity?

For unlimited access to Homework Help, a Homework+ subscription is required.

Trinidad Tremblay
Trinidad TremblayLv2
13 Jun 2018

Unlock all answers

Get 1 free homework help answer.
Already have an account? Log in

Related textbook solutions

Related questions

Related Documents

Weekly leaderboard

Start filling in the gaps now
Log in